Download as pdf or txt
Download as pdf or txt
You are on page 1of 34

Differential Equation With Boundary-Value Problems by D.G.Zill and M.R.

Cullen
EXERCISES 3.1 (Modeling with First-Order Differential Equations)

Problem 1 The population of a community is known to increase at a rate proportional to the number of people present at time t. If an
initial population P0 has doubled in 5 years, how long will it take to triple? To quadruple?
Problem 2 Suppose it is known that the population of the community in Problem 1 is 10,000 after 3 years. What was the initial
population P0 ? What will be the population in 10 years? How fast is the population growing at t = 10?
Solution (Problem 1):
Let P = P(t ) be the population at time t (continuation)
P0 = the initial population When P = 4 P0 :
Solving the differential equation,
4 P0 = P0e
( 1
5 )
ln 2 t

dP
P
dt
4=e
( 1
5 )
ln 2 t

dP
 = kP
dt
 ln 4 = ln[e
( 1
5
ln 2 t )]
1

( )
 dP = k dt 1
P  ln 4 = ln 2 t
 E
1
P
dP = k dt E 5
4 ln 3
t =
 ln P = kt + c ln 2
 e ln P = e kt + c  t  10 years
 P = ec e kt
Solution (Problem 2):
 P = P(t ) = ce kt
From Problem 1, recall that P = P0e
( )t .
1
5
ln 2

At t = 0 years, P = P0 :
Given P = 10,000 at t = 3 years, solving for the initial
P0 = cek (0)  c = P0 population:
So,
kt  10,000 = P0e
( 1
5 )
ln 2 (3)

P = P0e
3
− ln 2
 P0 = 10,000e 5

At t = 5 years, P = 2 P0 :
 P0 = 6,597.5
2 P0 = P0e k (5)  P0  6,598
 2 = e5k
 ln 2 = ln(e5k ) At t = 10 years:
 ln 2 = 5k
P = P(10) = 6,598e
( 1
5 )
ln 2 (10)

1
 k = ln 2  P(10) = 26,392
5
So,

P = P(t ) = P0e
( )t
1
5
ln 2 The rate at which the population is growing at t = 10 years is
P = kP
1
When P = 3P0 :  P = P(10) = (ln 2)(26,392) = 3,658.7
5

3 P0 = P0e
( )t
1
5
ln 2  P(10)  3,659 persons per year

3=e
( 1
5 )
ln 2 t

 ln 3 = ln[e
( 1
5 )]
ln 2 t

 ln 3 =
1
5 ( )
ln 2 t

5ln 3
t =
ln 2
 t  7.9 years

This document is not guaranteed by considering the items would reflect everything during your exam. But it will help you to understand,
nevertheless, on how the solution thoroughly works, provided that the methods are applied on the reference. The final answers are majority
replicated from the solution manual. Should be informed to me, typographical errors that may exist.
Differential Equation With Boundary-Value Problems by D.G.Zill and M.R.Cullen
EXERCISES 3.1 (Modeling with First-Order Differential Equations)

Problem 3 The population of a town grows at a rate proportional to the population present at time t. The initial population of 500
increases by 15% in 10 years. What will be the population in 30 years? How fast is the population growing at t = 30?
Solution:
Let P = P(t ) be the population at time t
P0 = the initial population
Solving the differential equation,
dP
P
dt
dP
 = kP
dt
1
 dP = k dt
P
 E 1
P
dP = k dt E
 ln P = kt + c
 e ln P = e kt + c
 P = ec e kt
 P = P(t ) = ce kt

At t = 0 years, P = 500 :
500 = cek (0)  c = 500
So,
P = 500ekt

At t = 10 years, P = 500 + 500(0.75) = 575 :


575 = 500e k (10)
575
 = e10 k
500
575
 ln = ln(e10 k )
500
575
 ln = 10 k
500
1 575
 k = ln
10 500
So,

P = P(t ) = 500e
( 1
10
ln
575
500 )
t

At t = 30 years,

P = P(30) = 500e
( 1
10
ln
575
500 )
(30)

 P(30) = 760

The rate at which the population is growing at t = 30 years is


P = kP

 P = P(30) = ( 1 575
ln
10 500 )
(760) = 10.62

 P(30)  11 persons per year

This document is not guaranteed by considering the items would reflect everything during your exam. But it will help you to understand,
nevertheless, on how the solution thoroughly works, provided that the methods are applied on the reference. The final answers are majority
replicated from the solution manual. Should be informed to me, typographical errors that may exist.
Differential Equation With Boundary-Value Problems by D.G.Zill and M.R.Cullen
EXERCISES 3.1 (Modeling with First-Order Differential Equations)

Problem 4 The population of bacteria in a culture grows at a rate proportional to the number of bacteria present at time t. After 3 hours
it is observed that 400 bacteria are present. After 10 hours 2000 bacteria are present. What was the initial number of bacteria?
Solution:
Let P = P(t ) be the population of bacteria at time t (continuation)
P0 = the initial population Equating k to solve for P0 :
Solving the differential equation, 1 400 1 2,000
ln = ln
dP 3 P0 10 P0
P
dt 10 400 2,000
 ln = ln
dP 3 P0 P0
 = kP

( )
dt 10 3

1 400 2,000
 dP = k dt  ln = ln
P P0 P0

 E
1
P
dP = k dt E  40010 3   2,000 
 e  ln 10 3  = e  ln
P

P0 
 0  
 ln P = kt + c
40010 3 2,000
 e ln P = e kt + c  10 3 =
P0 P0
 P = ec e kt
40010 3
 P = P(t ) = ce kt  = P0 (10 3) −1
2,000
40010 3
At t = 0 hours, P = P0 :  = P0 7 3
2,000

( )
P0 = cek (0)  c = P0 37
40010 3
( )
37
So,  = P0 7 3
2,000
P = P0ekt
 P0 = 200.7
 P0  201
When P = 400 at t = 3 hours, and P = 2,000 at t = 10 hours, two
equations in terms of k are involved:
400 = P0 e k (3)
400
 = e3 k
P0
400
 ln = ln(e3k )
P0
400
 ln = 3k
P0
1 400
 k = ln
3 P0
and
2,000 = P0 e k (10)
2,000 10 k
 =e
P0
2,000
 ln = ln(e10 k )
P0
2,000
 ln = 10 k
P0
1 2,000
 k = ln
10 P0

This document is not guaranteed by considering the items would reflect everything during your exam. But it will help you to understand,
nevertheless, on how the solution thoroughly works, provided that the methods are applied on the reference. The final answers are majority
replicated from the solution manual. Should be informed to me, typographical errors that may exist.
Differential Equation With Boundary-Value Problems by D.G.Zill and M.R.Cullen
EXERCISES 3.1 (Modeling with First-Order Differential Equations)

Problem 5 The radioactive isotope of lead, Pb-209, decays at a rate proportional to the amount present at time t and has a half-life of
3.3 hours. If 1 gram of this isotope is present initially, how long will it take for 90% of the lead to decay?
Solution:
Let A = A(t ) be the amount of lead present at time t
A0 = the initial amount
Solving the differential equation,
dA
A
dt
dA
 = kA
dt
1
 dA = k dt
A
 E 1
A
dA = k dt E
 ln A = kt + c
 e ln A = e kt + c
 A = ec e kt
 A = A(t ) = ce kt

At t = 0 hours, A = 1 g:
1 = cek (0)  c = 1
So,
A = ekt

At t = 3.3 hours, A = 0.5 g (in half-life):


0.5 = e k (3.3)
 ln 0.5 = ln(e3.3k )
 ln 0.5 = 3.3k
ln 0.5
k=
3.3
So,
ln 0.5
t
A=e 3.3

When 90% of the lead has decayed,


1 − 1(0.90) = 0.1 g will remain. Solving for the time t,
ln 0.5
t
0.1 = e 3.3

ln 0.5
t
 ln 0.1 = ln[e 3.3 ]
ln 0.5
 ln 0.1 = t
3.3
ln 0.1
 t = 3.3
ln 0.5
 t  10.96 hours

This document is not guaranteed by considering the items would reflect everything during your exam. But it will help you to understand,
nevertheless, on how the solution thoroughly works, provided that the methods are applied on the reference. The final answers are majority
replicated from the solution manual. Should be informed to me, typographical errors that may exist.
Differential Equation With Boundary-Value Problems by D.G.Zill and M.R.Cullen
EXERCISES 3.1 (Modeling with First-Order Differential Equations)

Problem 6 Initially 100 milligrams of a radioactive substance was present. After 6 hours the mass had decreased by 3%. If the rate of
decay is proportional to the amount of the substance present at time t, find the amount remaining after 24 hours.
Problem 7 Determine the half-life of the radioactive substance described in Problem 6.
Solution:
Let A = A(t ) be the amount present at time t (continuation)
A0 = the initial amount At t = 24 hours:
Solving the differential equation,
A = A(24) = 100e
( 1
ln
6 100
97
)
(24)

dA
A  A(24) = 88.5 milligrams
dt
dA
 = kA
dt When the amount is in half-life, solving for the time t:
1
 dA = k dt
50 = 100e
( 1
ln
6 100
97
t)
A
 E 1
A
dA = k dt E 
50 ( ln ) t
1

= e 6 100
97

100
 ln A = kt + c
 e ln A = e kt + c  ln 0.5 = ln[e
( 1
ln
6 100
97
t )]
 A = ec e kt
 A = A(t ) = ce kt
 ln 0.5 = (
1 97
ln
6 100
t )
ln 0.5
t =
At t = 0 hours, A = 100 mg: (1 6)ln(97 100)
100 = cek (0)  c = 100  t = 136.5 hours
So,
A = 100ekt

At t = 6 hours, A = 100 − 100(0.03) = 97 mg:


97 = 100e k (6)
97
 = e6 k
100
97
 ln = ln[e6 k ]
100
97
 ln = 6k
100
1 97
 k = ln
6 100
So,

A = A(t ) = 100e
( 1
ln
6 100
97
)t

This document is not guaranteed by considering the items would reflect everything during your exam. But it will help you to understand,
nevertheless, on how the solution thoroughly works, provided that the methods are applied on the reference. The final answers are majority
replicated from the solution manual. Should be informed to me, typographical errors that may exist.
Differential Equation With Boundary-Value Problems by D.G.Zill and M.R.Cullen
EXERCISES 3.1 (Modeling with First-Order Differential Equations)

Problem 8 (a) Consider the initial-value problem dA dt = kA, A(0) = A0 as the model for the decay of a radioactive substance. Show
that, in general, the half-life T of the substance is T = − (ln2) k . (b) Show that the solution of the initial-value problem in part (a) can be
written A(t ) = A0 2−t T . (c) If a radioactive substance has the half-life T given in part (a), how long will it take an initial amount A0 of the
substance to decay to 1
8
A0 ?
Solution (a):
Let A = A(t ) be the amount present at time t (continuation)
A0 = the initial amount Solution (b):
Solving the differential equation, ln 2 ln 2
Recall that T = −  k=−
dA k T
A Thus,
dt
ln 2
dA − t
 = kA A = A0 e T
dt −t T

1 = A0 e ln(2 )

 dA = k dt
A A = A(t ) = A0 2 − t T
 E
1
A
dA = k dt E
 ln A = kt + c Solution (c):
When A = 18 A0 , solving for the time t:
 e ln A = e kt + c
1
 A = ec e kt A0 = A0 2 − t T
8
 A = A(t ) = ce kt  2 −3 = 2 − t T
−t
At t = 0, A = A0 :  −3 =
T
A0 = cek (0)  c = A0  −t = −3T
So,  t = 3T
A = A0ekt (Thus, the amount A0 will decay to 1
A0 in three half-lives.)
8

Solving for the time when amount is in half-life T, where


A = 12 A0 :
1
A0 = A0 e kt
2
1
 = e kt
2
 2 = e − kt
 ln 2 = ln[e − kt ]
 ln 2 = − kt
ln 2
t = T =−
k
(Note: The symbol T denotes only when the amount is in half-
life according to the problem.)

This document is not guaranteed by considering the items would reflect everything during your exam. But it will help you to understand,
nevertheless, on how the solution thoroughly works, provided that the methods are applied on the reference. The final answers are majority
replicated from the solution manual. Should be informed to me, typographical errors that may exist.
Differential Equation With Boundary-Value Problems by D.G.Zill and M.R.Cullen
EXERCISES 3.1 (Modeling with First-Order Differential Equations)

Problem 13 A thermometer is removed from a room where the temperature is 70° F and is taken outside, where the air temperature is
10° F. After one-half minute the thermometer reads 50° F. What is the reading of the thermometer at t = 1 min? How long will it take for
the thermometer to reach 15° F?
Solution:
Solving the differential equation, (continuation)
dT At t = 1 2 mins, T = 50 F :
 T − Tm (where Tm = 10 F)
dt 50 = 10 + 60e k (1 2)
dT
 = k (T − 10) 50 − 10
dt  = ek 2
1 60
 dT = k dt 2
T − 10  ln = ln[e k 2 ]
 E 1
T − 10
E
dT = k dt (Let u = T − 10  du = dT )
3
2 k
 ln =
 E 1
u
du = kt + c
3 2
 k = 2 ln
2
 ln u = kt + c 3
So,
 ln T − 10 = kt + c
ln T −10 T = 10 + 60e
( 2
2 ln
3 )t
e = e kt + c
 T − 10 = ec ekt
At t = 1 min,
 T − 10 = cekt
kt T = T (1) = 10 + 60e
( 2 ln
2
3)(1)
 T = T (t ) = 10 + ce
 T (1) = 36.7 F
At t = 0 mins, T = 70 F :
70 = 10 + cek (0) When T = 15 F,
 c = 70 − 10  c = 60
So, 15 = 10 + 60e
( 2 ln
2
3
t)
T = 10 + 60ekt

15 − 10 ( 2 ln ) t
=e 3
2

60

 ln
1 ( 2 ln ) t
= ln[e 3 ]
2

12
1
( )
 ln = 2 ln t
12
2
3
ln(1 12)
t =
2 ln(2 3)
 t = 3.06 minutes

This document is not guaranteed by considering the items would reflect everything during your exam. But it will help you to understand,
nevertheless, on how the solution thoroughly works, provided that the methods are applied on the reference. The final answers are majority
replicated from the solution manual. Should be informed to me, typographical errors that may exist.
Differential Equation With Boundary-Value Problems by D.G.Zill and M.R.Cullen
EXERCISES 3.1 (Modeling with First-Order Differential Equations)

Problem 14 A thermometer is taken from an inside room to the outside, where the air temperature is 5° F. After 1 minute the
thermometer reads 55° F, and after 5 minutes it reads 30° F. What is the initial temperature of the inside room?
Solution:
Solving the differential equation, (continuation)
dT By using again Eq.1 and Eq.2 in terms of e5k , since from Eq.1:
 T − Tm (where Tm = 5 F)

( )
dt 50 50
5

dT = ek  = e5 k ,
 = k (T − 5) c c
dt
we equate to solve for c:
1

( )
 dT = k dt 50
5
25
T −5 =
 E 1
T −5
E
dT = k dt (Let u = T − 5  du = dT )
c
50 5
c
25
 5 =
 E 1
u
du = kt + c c
505
c

 ln u = kt + c  = c 5 −1
25
 ln T − 5 = kt + c 505
 = c4
ln T − 5 25
e = ekt + c

( )
14
 T − 5 = ece kt 505
c=
25
 T − 5 = ce kt
Thus,
 T = T (t ) = 5 + ce kt

( ) ( )
14 1
505 ln 0.5 t
T = T (t ) = 5 + e 4

At t = 1 min, T = 55 F : 25
55 = 5 + cek (1)
55 − 5 The initial temperature is

( )
 =c
ek T = T (0) = 5 +
505
14

e
( 1
4 )
ln 0.5 (0)

50 25
 k =c (Eq.1)
e  T (0) = 64.46 F

At t = 5 mins, T = 30 F :
30 = 5 + cek (5)
30 − 5
 = e5 k
c
25
 5k = c (Eq.2)
e

Equating Eq.1 and Eq.2 to solve for k,


50 25
=
e k e5k
25
 e5k − k =
50
25
 e5k − k =
50
 e 4 k = 0.5
 ln[e 4 k ] = ln 0.5
 4 k = ln 0.5
1
k= ln 0.5
4

This document is not guaranteed by considering the items would reflect everything during your exam. But it will help you to understand,
nevertheless, on how the solution thoroughly works, provided that the methods are applied on the reference. The final answers are majority
replicated from the solution manual. Should be informed to me, typographical errors that may exist.
Differential Equation With Boundary-Value Problems by D.G.Zill and M.R.Cullen
EXERCISES 3.1 (Modeling with First-Order Differential Equations)

Problem 15 A small metal bar, whose initial temperature was 20° C, is dropped into a large container of boiling water. How long will it
take the bar to reach 90° C if it is known that its temperature increases 2° C in 1 second? How long will it take the bar to reach 98° C?
Solution:
The boiling temperature for water is 100° C. (continuation)
So, assume Tm = 100 C. Thus,
Solving the differential equation,
T = T (t ) = 100 − 80e
( )t
ln
39
40
dT
 T − Tm
dt
dT When T = 90 C , solving for the time t:
 = k (T − 100)
dt
90 = 100 − 80e
( )t
ln
39
40
1
 dT = k dt
T − 100

90 − 100 ( ln ) t
= e 40
39

 E 1
E
dT = k dt (Let u = T − 100  du = dT )
−80
T − 100 1 ( ln ) t
39

 E 1
du = kt + c
 ln = ln[e 40 ]
8

( )
u 1 39
 ln u = kt + c  ln = ln t
8 40
 ln T − 100 = kt + c
ln(1 8)
ln T −100 t =
e = e kt + c ln(39 40)
 T − 100 = ec e kt  t = 82.13 seconds
 T − 100 = ce kt
 T = T (t ) = 100 + ce kt When T = 98 C , solving for the time t:

98 = 100 − 80e
( )t
ln
39
40

At t = 0 s, T = 20 C :
20 = 100 + ce k (0) 
98 − 100 ( ln ) t
= e 40
39

−80
 c = 20 − 100  c = −80
So,  ln
1 ( ln ) t
= ln[e 40 ]
39

T = 100 − 80ekt 40

At t = 1 s, T = 22 C :
 ln
1
40 ( )
= ln
39
40
t

22 = 100 − 80e k (1) ln(1 40)


t =
22 − 100 ln(39 40)
 = ek
−80  t = 145.70 seconds
39
 ln = ln[e k ]
40
39
 k = ln
40

This document is not guaranteed by considering the items would reflect everything during your exam. But it will help you to understand,
nevertheless, on how the solution thoroughly works, provided that the methods are applied on the reference. The final answers are majority
replicated from the solution manual. Should be informed to me, typographical errors that may exist.
Differential Equation With Boundary-Value Problems by D.G.Zill and M.R.Cullen
EXERCISES 3.1 (Modeling with First-Order Differential Equations)

Problem 16 Two large containers A and B of the same size are filled with different fluids. The fluids in containers A and B are
maintained at 0° C and 100° C, respectively. A small metal bar, whose initial temperature is 100° C, is lowered into container A. After
1 minute the temperature of the bar is 90° C. After 2 minutes the bar is removed and instantly transferred to the other container. After
1 minute in container B the temperature of the bar rises 10° C. How long, measured from the start of the entire process, will it take the
bar to reach 99.9° C?
Solution:
For first container, Tm1 = 0 C. (continuation)
Solving the differential equation,
dT1
 E
1
u
E
du = k2 dt
 T1 − Tm1
dt  ln u = k2 t + c2
dT1  ln T2 − 100 = k2 t + c2
 = k1 (T1 − 0)
dt ln T −100
e 2 = e k2 t + c2
1
 dT1 = k1 dt
T1  T2 − 100 = ec2 e k2 t

 E T1 dT = k E dt
1
1 1
 T2 = T2 (t ) = 100 + c2 e k2 t

 ln T1 = k1t + c1 When the metal bar is immersed in the second container,


e ln T1
=e k1t + c1 T2 = 81 C at t = 0 by starting the next process.

 T1 = ec1 e k1t 81 = 100 + c2ek2 (0)  c2 = 81 − 100  c2 = −19


So,
 T1 = T1 (t ) = c1e k1t
T2 = 100 − 19ek2 t
At t = 0 mins, T1 = 100 C (the initial temperature of the metal
At t = 1 min, T2 = 81 + 10 = 91 C :
bar):
100 = c1ek1 (0)  c1 = 100 91 = 100 − 19e k2 (1)
So, 91 − 100
 = e k2
T1 = 100ek1t −19
9
 ln = ln[e k2 ]
19
At t = 1 min, T1 = 90 C :
9
90 = 100e k1 (1)  k2 = ln
19
90 So,
 = ek1
100
T2 = 100 − 19e
( )t
ln
9
19
9
 ln = ln[ek1 ]
10
9 When T2 = 99.9 C , solving for the time t:
 k1 = ln

So,
10
99.9 = 100 − 19e
( )t
ln
9
19

T1 = 100e
( )t
ln
9
10

99.9 − 100 ( ln ) t
= e 19
9

−19
At t = 2 mins:  ln
0.1 ( ln ) t
= ln[e 19 ]
9

( ) (2) = 81 C
9 19

( )
ln
T1 = T1 (2) = 100e 10
0.1 9
 ln = ln t
19 19
For second container, Tm2 = 100 C. ln(0.1 19)
t =
Solving the differential equation, ln(9 19)
dT2  t = 7.02 minutes
 T2 − Tm 2
dt
dT Thus, the total time of the entire process is
 2 = k2 (T2 − 100)
dt t = 2 + 7.02
1  t = 9.02 minutes
 dT2 = k2 dt
T2 − 100

 E 1
T2 − 100
E
dT2 = k2 dt (Let u = T2 − 100  du = dT2 )

This document is not guaranteed by considering the items would reflect everything during your exam. But it will help you to understand,
nevertheless, on how the solution thoroughly works, provided that the methods are applied on the reference. The final answers are majority
replicated from the solution manual. Should be informed to me, typographical errors that may exist.
Differential Equation With Boundary-Value Problems by D.G.Zill and M.R.Cullen
EXERCISES 3.1 (Modeling with First-Order Differential Equations)

Problem 17 A thermometer reading 70° F is placed in an oven preheated to a constant temperature. Through a glass window in the oven
door, an observer records that the thermometer reads 110° F after minute and 145° F after 1 minute. How hot is the oven?
Solution:
Solving the differential equation, (continuation)
dT In terms of ek 2 , Eq.1 and Eq.2 becomes
 T − Tm
dt 110 − Tm
dT = ek 2
 = k (T − Tm ) 70 − Tm
dt and

( )
1
 dT = k dt 145 − Tm 145 − Tm
12
T − Tm = ek  = ek 2
 E 1
T − Tm
E
dT = k dt (Let u = T − Tm  du = dT )
70 − Tm
Equating to solve for Tm :
70 − Tm

E
1
( )
12
 du = kt + c 110 − Tm 145 − Tm
u =
70 − Tm 70 − Tm
 ln u = kt + c

( )
2
 ln T − Tm = kt + c 110 − Tm 145 − Tm
 =
ln T −Tm 70 − Tm 70 − Tm
e = e kt + c
(110 − T ) 2
145 − Tm
 T − Tm = ec e kt  m
=
(70 − Tm ) 2
70 − Tm
 T − Tm = ce kt
(110 − Tm )2 (70 − Tm )
 T = T (t ) = Tm + ce kt  = 145 − Tm
(70 − Tm )2
(110 − Tm )2
At t = 0 min, T = 70 F :  = 145 − Tm
70 − Tm
70 = Tm + ce k (0) (110 − Tm )(110 − Tm )
 c = 70 − Tm  = 145 − Tm
70 − Tm
So,  (110 − Tm )(110 − Tm ) = (145 − Tm )(70 − Tm )
T = Tm + (70 − Tm )ekt  12100 − 110Tm − 110Tm + Tm 2 = 10150 − 145Tm − 70Tm + Tm 2
 12100 − 220Tm + Tm 2 = 10150 − 215Tm + Tm 2
At t = 1 2 mins, T = 110 F and at t = 1 min, T = 145 F ; two
 12100 − 220Tm = 10150 − 215Tm
equations are involved:
 −220Tm + 215Tm = 10150 − 12100
110 = Tm + (70 − Tm )ek (1 2) (Eq.1)
 −5Tm = −1950
k (1)
145 = Tm + (70 − Tm )e (Eq.2)
−1950
 Tm =
−5
 Tm = 390 F

This document is not guaranteed by considering the items would reflect everything during your exam. But it will help you to understand,
nevertheless, on how the solution thoroughly works, provided that the methods are applied on the reference. The final answers are majority
replicated from the solution manual. Should be informed to me, typographical errors that may exist.
Differential Equation With Boundary-Value Problems by D.G.Zill and M.R.Cullen
EXERCISES 3.1 (Modeling with First-Order Differential Equations)

Problem 19 A dead body was found within a closed room of a house where the temperature was a constant 70° F. At the time of
discovery, the core temperature of the body was determined to be 85° F. One hour later a second measurement showed that the core
temperature of the body was 80° F. Assume that the time of death corresponds to t = 0 and that the core temperature at that time was
98.6° F. Determine how many hours elapsed before the body was found. [Hint: Let t1  0 denote the time that the body was discovered.]
Solution:
Solving the differential equation, (continuation)
dT Equating to solve for k,
 T − Tm (where Tm = 70 F)
dt 15 10 − k
= e
dT 28.6 28.6
 = k (T − 70)
dt 10
 ek =
1 15
 dT = k dt
T − 70 k 2
 ln[e ] = ln
 E 1
T − 70
E
dT = k dt (Let u = T − 70  du = dT )
2
3
 k = ln
 E 1
u
du = kt + c 3

 ln u = kt + c From Eq.1, solving for t1 :


 ln T − 70 = kt + c 15
e kt1 =
ln T − 70 kt + c 28.6
e =e
15
 T − 70 = ec e kt  ln[e kt1 ] = ln
28.6
 T − 70 = ce kt 15
 kt1 = ln
 T = T (t ) = 70 + ce kt 28.6
1 15
 t1 = ln
At t = 0 mins, T = 98.6 F : k 28.6
98.6 = 70 + ce k (0) ln(15 28.6)
 t1 =
ln(2 3)
 c = 98.6 − 70  c = 28.6
So,  t1 = 1.6 hours
T = 70 + 28.6ekt (about 1.6 hours prior to the discovery of the body)

If t1  0 be the time of discovery of the body, then


T (t1 ) = 70 + 28.6ekt1 = 85 (Eq.1)
T (t1 + 1) = 70 + 28.6ek (t1 +1) = 80 (Eq.2)
In terms of ekt1 , Eq.1 and Eq.2 becomes
85 − 70 15
ekt1 =  ekt1 =
28.6 28.6
and
80 − 70
e k ( t1 +1) =
28.6
kt1 k 10
e e =
28.6
10 − k
 e kt1 = e
28.6

This document is not guaranteed by considering the items would reflect everything during your exam. But it will help you to understand,
nevertheless, on how the solution thoroughly works, provided that the methods are applied on the reference. The final answers are majority
replicated from the solution manual. Should be informed to me, typographical errors that may exist.
Differential Equation With Boundary-Value Problems by D.G.Zill and M.R.Cullen
EXERCISES 3.1 (Modeling with First-Order Differential Equations)

Problem 20 The rate at which a body cools also depends on its exposed surface area S. If S is a constant, then a modification of
dT dt = k (T − Tm ) is
dT
= kS (T − Tm ),
dt
where k  0 and Tm is a constant. Suppose that two cups A and B are filled with coffee at the same time. Initially, the temperature of the
coffee is 150° F. The exposed surface area of the coffee in cup B is twice the surface area of the coffee in cup A. After 30 min the
temperature of the coffee in cup A is 100° F. If Tm = 70° F, then what is the temperature of the coffee in cup B after 30 min?
Solution:
Solving the differential equation,
dT
= kS (T − Tm )
dt
1
 dT = kS dt
T − Tm

 E 1
T − Tm
E
dT = kS dt (Let u = T − Tm  du = dT )

 E
1
u
du = kSt + c

 ln u = kSt + c
 ln T − Tm = kSt + c
ln T −Tm
e = e kSt + c
 T − Tm = ec e kSt
 T − Tm = ce kSt
 T = T (t ) = Tm + ce kSt

At t = 1 min, T = T0 :
T0 = Tm + ce kS (0)
 c = T0 − Tm  c = 150 − 70  c = 80 F
So,
T = 70 + 80ekSt

For cup A,
TA = 70 + 80ekSt
and for cup B (which is twice the surface area S in cup A),
TB = 70 + 80e2kSt

At t = 30 mins, TA = 100 F (which becomes twice the surface area S in cup A):
100 = 70 + 80e kS (30)
100 − 70
 e30 kS =
80
3
 e30 kS =
8

At t = 30 mins, whereas in cup B, the temperature is


TB = 70 + 80(ekSt )2
 TB = 70 + 80(e30 kS )2

()
2
3
 TB = 70 + 80
8
 TB = 81.25 F

This document is not guaranteed by considering the items would reflect everything during your exam. But it will help you to understand,
nevertheless, on how the solution thoroughly works, provided that the methods are applied on the reference. The final answers are majority
replicated from the solution manual. Should be informed to me, typographical errors that may exist.
Differential Equation With Boundary-Value Problems by D.G.Zill and M.R.Cullen
EXERCISES 3.1 (Modeling with First-Order Differential Equations)

Problem 21 A tank contains 200 liters of fluid in which 30 grams of salt is dissolved. Brine containing 1 gram of salt per liter is then
pumped into the tank at a rate of 4 L/min; the well-mixed solution is pumped out at the same rate. Find the number A(t) of grams of salt
in the tank at time t.
Problem 22 Solve Problem 21 assuming that pure water is pumped into the tank.
Solution (Problem 21):
Model: (continuation)
Solution (Problem 22):
1 g of salt,
Model:
4 L/min
200 L, 0 g of salt,
30 g of salt 4 L/min 4 L/min
For input rate and output rate, 200 L,
30 g of salt 4 L/min
g L
min = 1  4 = 4 g min
L min For input rate and output rate,
and g L
min = 0  4 = 0 g min
A g L A L min
mout =  4 = g min
200 L min 50 and
A g L A
mout =  4 = g min
Solving the differential equation, 200 L min 50
dA
= min − mout
dt Solving the differential equation,
dA A dA
 =4− = min − mout
dt 50 dt
dA A  dA A
 + = 4 Solving for the  =0−
dt 50  dt 50
integrating factor,
( )
1 1
t 50 dA A
e + = e (4)
t 50
e
E 501 dt = e 501 E dt = e 50t  dA = − dt
A 50
dt 50
d t 50
 e A  = 4et 50
 E 1
A
dA = −
1
50
dt E
dt
1
 d et 50 A  = 4et 50 dt  ln A = − t + c
50
E E
 d et 50 A  = 4 et 50 dt  e ln A = e

1
50
t +c

1 t 50  A = ec e − t 50
 et 50 A = 4  e +c
1 50
 A = A(t ) = ce − t 50
 et 50 A = 200et 50 + c
 e − t 50 (et 50 A) = e − t 50 (200et 50 + c) At t = 0 mins, A = 30 g (of salt):
 A = A(t ) = 200 + ce − t 50
30 = ce−0 50  c = 30
Thus,
At t = 0 mins, A = 30 g (of salt): A(t ) = 30e − t 50
30 = 200 + ce −0 50
 c = 30 − 200  c = −170

Thus,
A(t ) = 200 − 170e − t 50

This document is not guaranteed by considering the items would reflect everything during your exam. But it will help you to understand,
nevertheless, on how the solution thoroughly works, provided that the methods are applied on the reference. The final answers are majority
replicated from the solution manual. Should be informed to me, typographical errors that may exist.
Differential Equation With Boundary-Value Problems by D.G.Zill and M.R.Cullen
EXERCISES 3.1 (Modeling with First-Order Differential Equations)

Problem 23 A large tank is filled to capacity with 500 gallons of pure water. Brine containing 2 pounds of salt per gallon is pumped
into the tank at a rate of 5 gal/min. The well-mixed solution is pumped out at the same rate. Find the number A(t) of pounds of salt in the
tank at time t.
Problem 24 In Problem 23, what is the concentration c(t) of the salt in the tank at time t? At t = 5 min? What is the concentration of the
salt in the tank after a long time, that is, as t →  ? At what time is the concentration of the salt in the tank equal to one-half this limiting
value?
Problem 25 Solve Problem 23 under the assumption that the solution is pumped out at a faster rate of 10 gal/min. When is the tank
empty?
Solution (Problem 23):
Model: (continuation)
At t = 5 mins,
2 lb/gal,
5 gal/min c(5) = 2 − 2e−5 100 = 0.0975 lb gal
500 g,
0 g of salt 5 gal/min When t → ,
For input rate and output rate,
(
c(t ) = lim 2 − 2e − t 100
t →
)
lb gal
min = 2  5 = 10 lb min  c = 2 − 2(0) = 2 lb gal
gal min
and
A lb gal A Since the limiting value of the concentration is
mout =  5 = lb min 2 lb gal, half of it is 1 lb gal. Solving for the time t,
500 gal min 100
1 = 2 − 2e − t 100
Solving the differential equation, 1− 2
 = e − t 100
dA −2
= min − mout
dt  2 = et 100
dA A  ln 2 = ln(et 100 )
 = 10 −
dt 100
t
dA A   ln 2 =
 + = 10  Solving for the 100
dt 100   t = 100 ln 2
integrating factor,
 et 100 (dA
+
dt 100
A
)= et 100 (10)
e
E 1
100
dt
1

= e 100
E dt = e t
100
 t = 69.31 minutes

d
 et 100 A  = 10et 100 Solution (Problem 25):
dt Model:
 d et 100 A  = 10et 100 dt
2 lb/gal,
E E
 d et 100 A  = 10 et 100 dt 5 gal/min
500 g,
1 t 100 10 gal/min
 et 100 A = 10  e +c 0 g of salt
1 100
The output rate in this case is
 et 100 A = 1000et 100 + c A A
 e − t 100 (et 100 A) = e − t 100 (1000et 100 + c)
lb gal = lb gal
500 − (10 − 5)t 500 − 5t
 A = A(t ) = 1000 + ce − t 100 In order the tank to be empty at a certain time t, the volume must
be 0 gal. Solving for the time t,
At t = 0 mins, A = 0 lb (no salt in pure water): 500 − 5t = 0
0 = 1000 + ce −0 100  −5t = −500
 c = 0 − 1000  c = −1000 −500
t =
Thus, −5
A(t ) = 1000 − 1000e − t 100  t = 100 minutes

Solution (Problem 24):


The concentration at time t is
A(t ) 1000 − 1000e− t 100
c (t ) = =
500 500
 c(t ) = 2 − 2e− t 100

This document is not guaranteed by considering the items would reflect everything during your exam. But it will help you to understand,
nevertheless, on how the solution thoroughly works, provided that the methods are applied on the reference. The final answers are majority
replicated from the solution manual. Should be informed to me, typographical errors that may exist.
Differential Equation With Boundary-Value Problems by D.G.Zill and M.R.Cullen
EXERCISES 3.1 (Modeling with First-Order Differential Equations)

Problem 27 A large tank is partially filled with 100 gallons of fluid in which 10 pounds of salt is dissolved. Brine containing 12 pound of
salt per gallon is pumped into the tank at a rate of 6 gal/min. The well-mixed solution is then pumped out at a slower rate of 4 gal/min.
Find the number of pounds of salt in the tank after 30 minutes.
Solution:
Model:
½ lb of salt
per gal,
6 gal/min 100 g,
10 g of salt 4 gal/min
For input rate and output rate,
1 lb gal
min =  6 = 3 lb min
2 gal min
and
A lb gal 4A
mout =  4 = lb min
100 − (4 − 6) gal min 100 + 2t

Solving the differential equation,


dA
= min − mout Solving for the integrating
dt factor,
dA 4A

dt
=3−
100 + 2t e
E 2
dt
= e 50 + t
50 + t
2 E 1
dt

dA 4A (Let u = 50 + t  du = dt )
 + =3
dt 2(50 + t )
=e
2 E 1
u
du
dA 2 
 + A = 3
dt 50 + t  = e2 ln u
 (50 + t )2 (
dA
+
dt 100
A
)
= (50 + t )2 (3) = e2 ln(50 + t )
2
d = e ln(50 + t )
 (50 + t )2 A  = 3(50 + t )2
dt = (50 + t )2
 d (50 + t )2 A  = 3(50 + t )2 dt

E E
 d (50 + t )2 A  = 3 (50 + t )2 dt (Let u = 50 + t  du = dt )

E
 (50 + t )2 A = 3 u 2 du
1
 (50 + t )2 A = 3  u3 + c
3
 (50 + t )2 A = u3 + c
 (50 + t )2 A = (50 + t )3 + c
 (50 + t )−2 (50 + t )2 A  = (50 + t )−2 (50 + t )3 + c 
 A = A(t ) = 50 + t + c(50 + t )−2

At t = 0 mins, A = 10 lb:
10 = 50 + 0 + c(50 + 0)−2
 10 − 50 = c(50)−2
 c = −40(50)2
 c = −100,000
Thus,
A(t ) = 50 + t − 100,000(50 + t )−2

At t = 30 mins,
A(30) = 50 + 30 − 100,000(50 + 30)−2
 A(30) = 64.38 lb

This document is not guaranteed by considering the items would reflect everything during your exam. But it will help you to understand,
nevertheless, on how the solution thoroughly works, provided that the methods are applied on the reference. The final answers are majority
replicated from the solution manual. Should be informed to me, typographical errors that may exist.
Differential Equation With Boundary-Value Problems by D.G.Zill and M.R.Cullen
EXERCISES 3.1 (Modeling with First-Order Differential Equations)

Problem 29 A 30-volt electromotive force is applied to an LR series circuit in which the inductance is 0.1 henry and the resistance is
50 ohms. Find the current i(t) if i(0) = 0. Determine the current as t → .
Solution:
Model:
0.1 H

30 V
50 Ω

Applying Kirchhoff’s voltage law


and solving the differential equation,
di
L + Ri = E
dt
di
 0.1 + 50i = 30
dt

1
0.1 ( di
0.1 + 50i =
dt ) 1
0.1
(30)

di  Solving for the


 + 500i = 300 
dt  integrating factor,

 e500 t
di
dt ( )
+ 500i = e500 t (300) e
E 500 dt = e500 E dt = e500t
d 500 t
 e i  = 300e500 t
dt
 d e500 t i  = 300e500 t dt

E E
 d e500 t i  = 300 e500 t dt
1 500 t
 e500 t i = 300  e +c
500
1 500 t
 e500 t i = 300  e +c
500
3
 e500 t i = e500 t + c
5
e −500 t
(e i ) = e −500 t
500 t

5 (
3 500 t
e +c )
3
 i = i(t ) = + ce −500 t
5

At t = 0 s, i = 0 amperes:
3
0 = + ce−500(0)
5
3 3
c=0−  c=−
5 5
Thus,
3 3 −500 t
i (t ) = − e
5 5

When t → ,
3 3 −500 t
i(t ) = lim
− e
5 5 t → ( )
3 3
 i = − (0) = 0.6 amperes
5 5

This document is not guaranteed by considering the items would reflect everything during your exam. But it will help you to understand,
nevertheless, on how the solution thoroughly works, provided that the methods are applied on the reference. The final answers are majority
replicated from the solution manual. Should be informed to me, typographical errors that may exist.
Differential Equation With Boundary-Value Problems by D.G.Zill and M.R.Cullen
EXERCISES 3.1 (Modeling with First-Order Differential Equations)

Problem 31 A 100-volt electromotive force is applied to an RC series circuit in which the resistance is 200 ohms and the capacitance is
10−4 farad. Find the charge q(t) on the capacitor if q(0) = 0. Find the current i(t).
Solution:
Model:
200 Ω

100 V
10−4 F

Applying Kirchhoff’s voltage law


and solving the differential equation,
dq 1
R + q=E
dt C
dq 1
 200 + −4 q = 100
dt 10

1
200 ( dq 1
200 + −4 q =
dt 10
1
200
(100))
dq  Solving for the
 + 50q = 0.5
dt  integrating factor,

 e50 t ( dq
dt )
+ 50q = e50 t (0.5) e
E 50 dt
=e
50 E dt = e 50t

d 50 t
 e q  = 0.5e50 t
dt 
 d e50 t q  = 0.5e50 t dt

E E
 d e50 t q  = 0.5 e50 t dt
1
 e50 t q = 0.5  e50 t + c
50
1 50 t
 e50 t q = e +c
100
 e −50 t (e50 t q ) = e −50 t
100(
1 50 t
e +c )
1
 q = q (t ) = + ce −50 t
100

At t = 0 s, q = 0 coulombs:
1
0= + ce −50(0)
100
1 1
c=0−  c=−
100 100
Thus,
1 1 −50 t
q (t ) = − e
100 100

Solving for i(t),


d
i (t ) = q (t ) =
dt
d
(1

1 −50 t
dt 100 100
e )
( )
0
d 1 1 d −50 t
 i (t ) = − (e )
dt 100 100 dt
1
 i (t ) = − (−50e −50 t )
100
1
 i(t ) = e −50 t
2

This document is not guaranteed by considering the items would reflect everything during your exam. But it will help you to understand,
nevertheless, on how the solution thoroughly works, provided that the methods are applied on the reference. The final answers are majority
replicated from the solution manual. Should be informed to me, typographical errors that may exist.
Differential Equation With Boundary-Value Problems by D.G.Zill and M.R.Cullen
EXERCISES 3.1 (Modeling with First-Order Differential Equations)

Problem 32 A 200-volt electromotive force is applied to an RC series circuit in which the resistance is 1000 ohms and the capacitance
is 5  10−6 farad. Find the charge q(t) on the capacitor if i(0) = 0.4 amperes. Determine the charge and current at t = 0.005 s. Determine
the charge as t → .
Solution:
Model:
1000 Ω (continuation)
Solving for i(t),

200 V
5  10−6 F
d
i (t ) = q ( t ) =
dt
d
(1
dt 1000
+ ce −200 t )
( )
0
d 1 d −200 t
 i (t ) = +c (e )
dt 1000 dt
Applying Kirchhoff’s voltage law
and solving the differential equation,  i(t ) = −200ce −200 t
dq 1
R + q=E
dt C At t = 0 s, i = 0.4 amperes:
dq 1 0.4 = −200ce−200(0)
 1000 + q = 200
dt 5  10 −6 0.4 1

( )
1 dq 1 1 c=  c=−
 1000 + q = (200) −200 500
1000 dt 5  10 −6 1000 Thus,
dq  Solving for the 1 −200 t 2
 + 200q = 0.2  i(t ) = −200  − e  i(t ) = e −200 t
dt  integrating factor, 500 5

 e200 t ( dq
dt )
+ 200q = e200 t (0.2) e
E 200 dt = e200 E dt = e200t and

q (t ) =
1

1 −200 t
e
d 1000 500
 e200 t q  = 0.2e200 t
dt
 d e200 t q  = 0.2e200 t dt At t = 0.005 s,

E E
 d e200 t q  = 0.2 e200 t dt q(0.005) =
1

1000 500
1 −200(0.005)
e = 2.64  10 −4 coulombs

1 200 t and
 e200 t q = 0.2  e +c
200 2
i(0.005) = e−200(0.005) = 0.147 amperes
1 200 t 5
 e200 t q = e +c
1000

(
 e −200 t (e200 t q ) = e −200 t
1 200 t
e +c ) When t → ,

 q = q (t ) =
1
1000

+ ce −200 t
1
q(t ) = lim −
1000 500
t → (
1 −200 t
e )
1000 1 1
q= − (0) = 0.001 coulombs
1000 500

This document is not guaranteed by considering the items would reflect everything during your exam. But it will help you to understand,
nevertheless, on how the solution thoroughly works, provided that the methods are applied on the reference. The final answers are majority
replicated from the solution manual. Should be informed to me, typographical errors that may exist.
Differential Equation With Boundary-Value Problems by D.G.Zill and M.R.Cullen
EXERCISES 3.1 (Modeling with First-Order Differential Equations)

Problem 33 An electromotive force


120, 0  t  20
E (t ) = 
 0, t  20
is applied to an LR series circuit in which the inductance is 20 henries and the resistance is 2 ohms. Find the current i(t) if i(0) = 0.
Solution:
Model: (continuation)
20 H Solving for c1 given t = 0, i = 0 :
0 = 60 + c1e − 0 10
120, 0  t  20  c1 = 0 − 60
E (t ) = 
 0, t  20 2Ω  c1 = −60

To be the function continuous, provided that


Applying Kirchhoff’s voltage law
t = 20, and i(t ) = 60 − 60e−t 10  i(20) = 60 − 60e−20 10 .
and solving the differential equation,
Solving for c2 :
di
L + Ri = E 60 − 60e − 20 10 = c2e − 20 10
dt
di 120, 0  t  20 60 − 60e − 20 10
 20 + 2i =   c2 =
dt  0, t  20 e − 20 10

( )
1 di  1 (120), 0  t  20 60 60e −2
 20 + 2i =  20 1  c2 = −2 − −2
t  20 e e
20 dt  20 (0),
 c2 = 60e2 − 60
di 1 60, 0  t  20  Solving for the
 + i= 
dt 10  0, t  20  integrating factor, Thus,
E E dt = e
( )
1 1 t
e (60), 0  t  20
t 10 dt
di 1 e 10
=e 10 10  60 − 60e− t 10 , 0  t  20
 et 10 + i =  t 10 i (t ) = 
dt 10  e (0), t  20
(60e − 60)e , t  20
2 − t 10

60e , 0  t  20
t 10
d
 et 10i  = 
dt  0, t  20
60et 10 dt , 0  t  20
 d et 10i  = 
 0 dt , t  20

E
 d et 10i  = 

E t 10
60 e dt , 0  t  20
 0 dt ,

E t  20

 1 t 10
60  e + c1 , 0  t  20
e i=
t 10
1 10
 t  20
 0(t ) + c2 ,
60e + c1 , 0  t  20
t 10

 et 10i = 
 c2 , t  20

e (60e + c1 ), 0  t  20
− t 10 t 10

 e − t 10 (et 10i ) = 

− t 10
e (c2 ), t  20
60 + c1e , 0  t  20
− t 10

 i = i (t ) = 
 c2 e ,
− t 10
t  20

This document is not guaranteed by considering the items would reflect everything during your exam. But it will help you to understand,
nevertheless, on how the solution thoroughly works, provided that the methods are applied on the reference. The final answers are majority
replicated from the solution manual. Should be informed to me, typographical errors that may exist.
Differential Equation With Boundary-Value Problems by D.G.Zill and M.R.Cullen
EXERCISES 3.1 (Modeling with First-Order Differential Equations)

Problem 34 Suppose an RC series circuit has a variable resistor. If the resistance at time t is given by R = k1 + k2t, where k1 and k2 are
known positive constants, then R dq dt + (1 C)q = E(t ) becomes
dq 1
+ q = E (t ).(k1 + k2t )
dt C
If E(t ) = E0 and q(0) = q0 , where E0 and q0 are constants, show that

( )
1 Ck2
k1
q(t ) = E0C + (q0 − E0C ) .
k1 + k2t
Solution:
Solving the differential equation, (continuation)
dq 1 Solving for c given q(0) = q0 ,
(k1 + k2 t ) + q = E0

( )
dt C 1 Ck2
1
1  dq 1  1 q0 = E0C + c
 (k1 + k2 t ) + q  = E0 k1 + k2 (0)
k1 + k2t  dt C  k1 + k2t
c
dq 1 E0  q0 = E0C +
 + q= k11 Ck2
dt C (k1 + k2t ) k1 + k2t
c
 dq 1   E0   q0 − E0C = 1 Ck2
 (k1 + k2t )1 Ck2  + q  = (k1 + k2t )1 Ck2   k1
 dt C ( k1 + k 2 t )   k1 + k2t 
 c = (q0 − E0C )k11 Ck2
d
 (k1 + k2 t ) q  = E0 (k1 + k2t )(1 Ck2 ) −1
1 Ck2

dt
Thus,

( )
 d (k1 + k2t )1 Ck2 q  = E0 (k1 + k2t )(1 Ck2 ) −1 dt 1 Ck2
1
E
 d (k1 + k2t ) 1 Ck2
E
q  = E0 (k1 + k2t ) (1 Ck2 ) −1
dt
q(t ) = E0C + (q0 − E0C )k11 Ck2
k1 + k2 t

( )
1 Ck2
k1
(Let u = k1 + k2 t  q(t ) = E0C + (q0 − E0C )
1 k1 + k2t
 du = k2 dt or du = dt )
k2

 (k1 + k2t )1 Ck2 q = E0 E(u (1 Ck2 ) −1


) ( )
1
k2
du
(Nothing follows in this exercise.)

 =
E
k
Eu 0

2
(1 Ck2 ) −1
du

E0  1 
 =  u(1 Ck2 ) −1+1  + c
k2  (1 Ck 2 ) − 1 + 1 
E0
 = ( k1 + k2t )1 Ck2  + c
1C
 (k1 + k2t )1 Ck2 q = E0C (k1 + k2t )1 Ck2 + c
(k1 + k2t )1 Ck2 q 1
 =  E0C (k1 + k2 t )1 Ck2 + c 
(k1 + k2t )1 Ck2
(k1 + k2 t )1 Ck2 
E0C (k1 + k2 t )1 Ck2 c
q= +
(k1 + k2 t )1 Ck2
(k1 + k2 t )1 Ck2

( )
1 Ck2
1
 q = q(t ) = E0C + c
k1 + k2t

This document is not guaranteed by considering the items would reflect everything during your exam. But it will help you to understand,
nevertheless, on how the solution thoroughly works, provided that the methods are applied on the reference. The final answers are majority
replicated from the solution manual. Should be informed to me, typographical errors that may exist.
Problem 3 A thermometer reading 18°F is brought into a room where the temperature is 70°F; 1 min later the thermometer reading is
31°F. Determine the temperature reading as a function of time and, in particular, find the temperature reading 5 minutes after the
thermometer is first brought into the room.
Solution:
Solving the differential equation,
dT
 T − Tm (where Tm = 70 F)
dt
dT
 = k (T − 70)
dt
1
 dT = k dt
T − 70
 E 1
T − 70
E
dT = k dt (Let u = T − 70  du = dT )

 E 1
u
du = kt + c

 ln u = kt + c
 ln T − 70 = kt + c
ln T − 70
e = e kt + c
 T − 70 = ec e kt
 T − 70 = ce kt
 T = T (t ) = 70 + ce kt

At t = 0 mins, T = 18 F :
18 = 70 + cek (0)  c = 18 − 70  c = −52

At t = 1 min, T = 31 F :
31 = 70 − 52e k (1)
31 − 70
 ek =
−52
3
 ln(e ) = ln
k

4
3
 k = ln
4

Thus,

T = T (t ) = 70 − 52e
( )t
ln
3
4

When t = 1 min, the temperature reading is

T = T (1) = 70 − 52e
( ) (1)
ln
3
4

 T (1) = 57.7° F

This document is not guaranteed by considering the items would reflect everything during your exam. But it will help you to understand,
nevertheless, on how the solution thoroughly works, provided that the methods are applied on the reference. The final answers are majority
replicated from the solution manual. Should be informed to me, typographical errors that may exist.
Problem 4 A thermometer reading 75°F is taken out where the temperature is 20°F. The reading is 30°F 4 min later. Find (a) the
temperature reading 7 min after the thermometer was brought outside and (b) the time taken for the reading to drop from 75°F to within
a half degree of the air temperature.
Solution:
Solving the differential equation, (continuation)
dT So,
 T − Tm (where Tm = 20 F)
dt ( 1 ln 2 ) t
T = T (t ) = 20 + 55e 4 11
dT
 = k (T − 20)
dt
1 When t = 7 mins, the temperature reading is
 dT = k dt
T − 20
1

T = T (7) = 20 + 55e 4 11
(
ln
2
)
(7)

 E 1
T − 20
E
dT = k dt (Let u = T − 20  du = dT )
 T (7) = 22.8 F

 E 1
u
du = kt + c
(b)
 ln u = kt + c When T = 20.5 F (half-degree of the air temperature),
 ln T − 20 = kt + c solving for the time t:

e
ln T − 20
= e kt + c 20.5 = 20 + 55e
( 1
ln
4 11
2
t )
 T − 20 = ec e kt
kt 
20.5 − 20 ( ln ) t
1

= e 4 11
2

 T − 20 = ce 55
 T = T (t ) = 20 + ce kt
 ln
1 ( ln ) t
1

= ln[e 4 11 ]
2

110
(a) 4 ln(1 110)
At t = 0 mins, T = 75 F : t =
ln(2 11)
75 = 20 + cek (0)  c = 75 − 20  c = 55
 t = 11.03 mins

At t = 4 mins, T = 30 F :
30 = 20 + 55e k (4)
30 − 20
 e4 k =
55
2
 ln(e ) = ln
4k

11
2
 4 k = ln
11
1 2
 k = ln
4 11

This document is not guaranteed by considering the items would reflect everything during your exam. But it will help you to understand,
nevertheless, on how the solution thoroughly works, provided that the methods are applied on the reference. The final answers are majority
replicated from the solution manual. Should be informed to me, typographical errors that may exist.
Problem 5 At 1:00 P.M., a thermometer reading 70°F is taken outside where the air temperature is −10°F (ten below zero). At
1:02 P.M., the reading is 26°F. At 1:05 P.M., the thermometer is taken back indoors, where the air is at 70°F. What is the temperature
reading at 1:09 P.M.?
Solution:
Solving the differential equation, (continuation)
dT So,
 T − Tm (where Tm = −10 F)
dt ( 1 ln 9 ) t
T = T (t ) = −10 + 80e 2 20
dT
 = k (T − (−10))
dt
1 At 1:05 PM (t = 5), the temperature reading is
 dT = k dt
T + 10 ( 12 ln 209 ) (5)
 E 1
T + 10
E
dT = k dt (Let u = T + 10  du = dT )
T (5) = − 10 + 80
 T (5) = 0.9 F
e

 E du = kt + c
1
u When the thermometer is taken back indoors where Tm = 70 F ,
 ln u = kt + c
T = T (t ) = 70 + ce
( 1
2
ln
9
20 )t
 ln T + 10 = kt + c
ln T +10
e = e kt + c Starting over at 1:05 PM (t = 0), T = 0.9 F :
 T + 10 = ec ekt
0.9 = 70 + ce
( 1
2
ln
9
20 )
(0)
 c = 0.9 − 70  c = −69.1
 T + 10 = cekt
 T = T (t ) = −10 + ce kt At 1:09 PM (t = 4), the temperature reading is

At 1:00 PM (t = 0), T = 70 F : T = T (4) = 70 − 69.1 e


( 1
2
ln )
9
20
(4)

70 = −10 + cek (0)  c = 70 + 10  c = 80  T (4) = 56° F

At 1:02 PM (t = 2), T = 26 F :


26 = −10 + 80e k (2)
26 + 10
 e2 k =
80
9
 ln(e2 k ) = ln
20
9
 2 k = ln
20
1 9
 k = ln
2 20

This document is not guaranteed by considering the items would reflect everything during your exam. But it will help you to understand,
nevertheless, on how the solution thoroughly works, provided that the methods are applied on the reference. The final answers are majority
replicated from the solution manual. Should be informed to me, typographical errors that may exist.
Problem 6 At 9 A.M., a thermometer reading 70°F is taken outdoors, where the temperature is 15°F. At 9:05 A.M., the thermometer
reading is 45°F. At 9:10 A.M., the thermometer is taken back indoors, where the temperature is fixed at 70°F. Find (a) the reading at
9:20 A.M. and (b) when the reading, to the nearest degree, will show the correct (70°F) indoor temperature.
Solution:
Solving the differential equation, (continuation)
dT So,
 T − Tm (where Tm = 15 F)
dt ( 1 ln 6 ) t
T = T (t ) = 15 + 55e 5 11
dT
 = k (T − 15)
dt
1 At 9:10 AM (t = 10) , the thermometer reading is
 dT = k dt
T − 15 ( 15 ln 116 ) (10)
 E 1
T − 15
E
dT = k dt (Let u = T − 15  du = dT )
T = T (10) = 15 + 55
 T (10) = 31.364° F
e

 E du = kt + c
1
u When the thermometer is taken back indoors where Tm = 70 F ,
 ln u = kt + c
T = T (t ) = 70 + ce
( 1
ln
5 11
6
t )
 ln T − 15 = kt + c
ln T −15
e = e kt + c Starting over at 9:10 AM (t = 0), T = 31.364 F :
 T − 15 = ec ekt
31.364 = 70 + ce
( 1
ln
5 11
6
(0) )  c = 31.364 − 70  c = −38.636
 T − 15 = cekt
 T = T (t ) = 15 + ce kt At 9:20 AM (t = 10), the temperature reading is

At 9:00 AM (t = 0), T = 70 F : T = T (10) = 70 − 38.636e


( 1
ln
5 11 )
6
(10)

70 = 15 + cek (0)  c = 70 − 15  c = 55  T (10) = 58.5° F

At 9:05 AM (t = 5), T = 45 F :


When T = 69.5 F (by reading to the nearest degree of
45 = 15 + 55e k (5) Tm = 70 F ), solving for the time t:
45 − 15
 e5 k =
55 69.5 = 70 − 38.636e
( 1
ln
5 11
6
t )
 ln(e5 k ) = ln
6

69.5 − 70 ( ln ) t
= e 5 11
1 6

11
−38.636
6
 5k = ln
11  ln
0.5 ( ln ) t
= ln[e 5 11 ]
1 6

1 6 38.636
 k = ln 5ln(0.5 38.636)
5 11 t=
ln(6 11)
 t = 35.86 mins  36 mins
or 9:46 AM (starting from 9:10 AM)

This document is not guaranteed by considering the items would reflect everything during your exam. But it will help you to understand,
nevertheless, on how the solution thoroughly works, provided that the methods are applied on the reference. The final answers are majority
replicated from the solution manual. Should be informed to me, typographical errors that may exist.
Problem 7 At 2:00 P.M., a thermometer reading 80°F is taken outside, where the air temperature is 20°F. At 2:03 P.M., the temperature
reading yielded by the thermometer is 42°F. Later, the thermometer is brought inside, where the air is at 80°F. At 2:10 P.M., the reading
is 71°F. When was the thermometer brought indoors?
Solution:
Solving the differential equation, (continuation)
dT So,
 T − Tm (where Tm = 20 F)
dt ( 1 ln 11 ) t
T = T (t ) = 20 + 60e 3 30
dT
 = k (T − 20)
dt
1 If the thermometer is brought inside at t0, it reads
 dT = k dt
T − 20 ( 1 ln 11 ) t0
T = T (t0 ) = 20 + 60e 3 30
 E 1
T − 20
E
dT = k dt (Let u = T − 20  du = dT ) and the new equation after taking inside where Tm = 80 F is

 E 1
u
du = kt + c ( 1 ln 11 ) t
T = T (t ) = 80 + ce 3 30
 ln u = kt + c
Starting over at 2:03 PM (t = 0),
 ln T − 20 = kt + c
e
ln T − 20
= e kt + c T (0) = 80 + ce
( 1 11
ln
3 30
(0) ) = 20 + 60e
( 1 11
ln t
3 30 0)
 T − 20 = ec e kt
 80 + c = 20 + 60e
( 1 11
ln t
3 30 0 )
kt
 T − 20 = ce
 T = T (t ) = 20 + ce kt  c = −60 + 60e
( 1 11
ln t
3 30 0 )

At 2:00 PM (t = 0), T = 80 F : At 2:10 PM (t = 10 − t0 ), T = 71 F, solving for t0:


80 = 20 + ce k (0)
 c = 80 − 20  c = 60
T (10 − t0 ) = 80 − 60 + 60e
( 1
ln
3 30
11
) t e(
0
1
ln
3 30
11
)(10 − t 0 )
= 71
At 2:03 PM (t = 3), T = 42 F :
 20 + 60e
( 10
3
ln
11
30 ) e( 2
− ln
3
11
30 )t 0
= 71
k (3)
42 = 20 + 60e
42 − 20 e
( 10
3
ln
11
30 ) e( 2
− ln
3
11
30 )t 0
=
71 − 20
 e3 k = 60
60
11 e
( −
2
3
ln
11
30 )t 0
=
51 ( −
e
10
3
ln
11
30 )
 ln(e3 k ) = ln
30 60

 3k = ln
11
 ln e
( −
2
3
ln
11
30 ) t  = ln  51 e(
0 −
10
3
ln
11
30 )
30    60 
1 11
 k = ln
(  51 e( −
) )
10 11
3 30 2 11 ln
 − ln t0 = ln 3 30

3 30  60 
ln
 51 e( −
10
3
ln
11
30 )
 60 
 t0 =
2 11
− ln
3 30
 t0 = 4.76 mins  5 mins
or 2:05 PM (starting from 2:00 PM)

This document is not guaranteed by considering the items would reflect everything during your exam. But it will help you to understand,
nevertheless, on how the solution thoroughly works, provided that the methods are applied on the reference. The final answers are majority
replicated from the solution manual. Should be informed to me, typographical errors that may exist.
Problem 10 For a substance C, the time rate of conversion is proportional to the square of the amount x of unconverted substance. Let k
be the numerical value of the constant of proportionality and let the amount of unconverted substance be x0 at time t = 0. Determine x
for all t ≥ 0.
Solution:
Analyzing the given problem where a simple chemical
conversion of a substance is involved:
Time Substance C Product
At initial time x0 0
At time t x0 − x x

Solving the differential equation,


dx
= −kx 2
dt
dx
 − 2 = k dt
x
E
 − x −2 dx = k dt E
−1
 −(− x ) = kt + c
1
 = kt + c
x
 x = x (t ) = (kt + c)−1

Solving for c, at t = 0, x = x0 :
1
x0 = (k (0) + c)−1  x0 = c −1  c =
x0

Thus, the amount of x at t ≥ 0 will be

( )
−1
1
x (t ) = kt +
x0

( )
−1
x0 kt + 1
 x (t ) =
x0
x0
 x (t ) =
x0 kt + 1

This document is not guaranteed by considering the items would reflect everything during your exam. But it will help you to understand,
nevertheless, on how the solution thoroughly works, provided that the methods are applied on the reference. The final answers are majority
replicated from the solution manual. Should be informed to me, typographical errors that may exist.
Problem 11 For a substance D, the time rate of conversion is proportional to the square root of the amount x of unconverted substance.
Let k be the numerical value of the constant of proportionality. Show that the substance will disappear in finite time and determine the
time.
Solution:
Analyzing the given problem where a simple chemical
conversion of a substance is involved:
Time Substance D Product
At initial time x0 0
At time t x0 − x x

Solving the differential equation,


dx
= −k x
dt
 x −1 2 dx = −k dt

E
 x −1 2 dx = −k dt E
1 12
 ( x ) = −kt + c
12
 2 x1 2 = − kt + c
−kt + c
 x1 2 =
2

( )
2
− kt
 x = x (t ) = +c
2

Solving for c, at t = 0, x = x0 :

( )
2
−k (0)
x0 == +c  x0 = c 2  c = x0
2

Solving for the time t, when the substance disappears:

( )
2
− kt
0= + x0
2
− kt
0= + x0
2
 0 = − kt + 2 x0
 kt = 2 x0
2
 t= x0
k

This document is not guaranteed by considering the items would reflect everything during your exam. But it will help you to understand,
nevertheless, on how the solution thoroughly works, provided that the methods are applied on the reference. The final answers are majority
replicated from the solution manual. Should be informed to me, typographical errors that may exist.
Problem 14 Radium decomposes at a rate proportional to the quantity of radium present. Suppose it is found that in 25 years
approximately 1.1% of a certain quantity of radium has decomposed. Determine approximately how long it will take for one-half the
original amount of radium to decompose.
Solution:
Let A = A(t ) be the amount of lead present at time t
A0 = the initial amount
Solving the differential equation,
dA
A
dt
dA
 = kA
dt
1
 dA = k dt
A
 E 1
A
dA = k dt E
 ln A = kt + c
 e ln A = e kt + c
 A = ec e kt
 A = A(t ) = ce kt

At t = 0, A = A0 :
A0 = cek (0)  c = A0

When t = 25 years, A = (1 − 0.011) A0 = 0.989 A0 :


0.989 A0 = A0ek (25)
 0.989 = e25k
 ln(0.989) = ln(e25k )
 ln(0.989) = 25k
1
 k = ln(0.989)
25

When A = 0.5 A0 (half-life), solving for time t:

0.5 A0 = A0 e
( 1
25
ln(0.989) t)
 0.5 = e
( 1
25
ln(0.989) t )
 ln(0.5) = ln[e
( 1
25 )]
ln(0.989) t

 ln(0.5) =
1
25 (
ln(0.989) t )
25ln(0.5)
t=
ln(0.989)
 t = 1566.65 years

This document is not guaranteed by considering the items would reflect everything during your exam. But it will help you to understand,
nevertheless, on how the solution thoroughly works, provided that the methods are applied on the reference. The final answers are majority
replicated from the solution manual. Should be informed to me, typographical errors that may exist.
Problem 15 A certain radioactive substance has a half-life of 38 hr. Find how long it takes for 90% of the radioactivity to be dissipated.
Solution:
Let A = A(t ) be the amount of lead present at time t
A0 = the initial amount
Solving the differential equation,
dA
A
dt
dA
 = kA
dt
1
 dA = k dt
A
 E 1
A
dA = k dt E
 ln A = kt + c
 e ln A = e kt + c
 A = ec e kt
 A = A(t ) = ce kt

At t = 0, A = A0 :
A0 = cek (0)  c = A0

When t = 38 hours, A = 0.5 A0 :


0.5 A0 = A0 e k (38)
 0.5 = e38k
 ln(0.5) = ln(e38 k )
 ln(0.5) = ln(e38 k )
 ln(0.5) = 38k
1
 k = ln(0.5)
38

When 90% are dissipated, A = 0.1A0 :

0.1A0 = A0 e
( 1
38
ln(0.5) t )
 0.1 = e
( 1
38
ln(0.5) t )
 ln(0.1) = ln[e
( 1
38 )]
ln(0.5) t

 ln(0.1) =
1
38 (
ln(0.5) t )
38ln(0.1)
t=
ln(0.5)
 t = 126.23 hours

This document is not guaranteed by considering the items would reflect everything during your exam. But it will help you to understand,
nevertheless, on how the solution thoroughly works, provided that the methods are applied on the reference. The final answers are majority
replicated from the solution manual. Should be informed to me, typographical errors that may exist.
Problem 16 A bacterial population B is known to have a rate of growth proportional to B itself. If between noon and 2 P.M. the
population triples, at what time, no controls being exerted, should B become 100 times what it was at noon?
Solution:
Solving the differential equation,
dB
B
dt
dB
 = kB
dt
1
 dB = k dt
B
 E
1
B
dB = k dt E
 ln B = kt + c
 e ln B = e kt + c
 B = ec e kt
 B = B(t ) = ce kt

At t = 0, B = B0 :
B0 = cek (0)  c = B0
So,
B = B0ekt

Between noon and 2 PM (t = 2) , B = 3B0 :


3B0 = B0e k (2)
 3 = e2 k
 ln 3 = ln(e2 k )
 ln 3 = 2 k
1
 k = ln 3
2

So,

B = B(t ) = B0e
( 1
2 )
ln 3 t

Solving for the time t at which population becomes


100 times the original.

100 B0 = B0e
( 1
2 )
ln 3 t

 100 = e
( 1
2 )
ln 3 t

 ln(100) = ln[e
( 1
2 )]
ln 3 t

 ln(100) = ( ) 1
2
ln 3 t

2 ln 3
t =
ln100
 t = 8.38 hours
 t = 8 hours and 22 minutes or 8:22 PM

This document is not guaranteed by considering the items would reflect everything during your exam. But it will help you to understand,
nevertheless, on how the solution thoroughly works, provided that the methods are applied on the reference. The final answers are majority
replicated from the solution manual. Should be informed to me, typographical errors that may exist.
Problem 24 A tank contains 80 gallons (gal) of pure water. A brine solution with 2 lb/gal of salt enters at 2 gal/min, and the well-stirred
mixture leaves at the same rate. Find (a) the amount of salt in the tank at any time and (b) the time at which the brine leaving will
contain 1 lb/gal of salt.
Problem 25 For the tank in Exercise 24, determine the limiting value for the amount of salt in the tank after a long time. How much
time must pass before the amount of salt in the tank reaches 80% of this limiting value?
Solution (Problem 24):
Model:
(continuation)
2 lb/gal,
2 gal/min
80 gal of (b)
pure water 2 gal/min If the concentration contains 1 lb/gal, the amount of salt will be
lb
A =1  80 gal = 80 lbs of salt
gal
(a)
For input rate and output rate,
Solving for the time t,
lb gal
min = 2  2 = 4 lb min 80 = 160 − 160e − t 40
gal min
80 − 160
and  = e − t 40
−160
A lb gal A
mout =  2 = lb min  0.5 = e − t 40
80 + (2 − 2)t gal min 40
 ln 0.5 = ln(e − t 40 )
Solving the differential equation, t
 ln 0.5 = −
dA 40
= min − mout  t = −40 ln 0.5 or 40ln2
dt
dA A  t = 27.73 mins
 =4−
dt 40
dA A  Solution (Problem 25):
 + = 4 Solving for the
dt 40  When t →  ,
integrating factor,
 et 40 (dA A
dt 40
+ ) = et 40 (4)
e
E 1
40
dt
1

= e 40
E dt = e t
40
A(t ) = lim (160 − 160e − t 40 )
t →

 A = 160 − 160(0)
d
 et 40 A  = 4et 40  A = 160 lb
dt
 d et 40 A  = 4et 40 dt
When the amount of salt in the tank reaches 80% of this limiting
E E
 d et 40 A  = 4 et 40 dt value, that is 0.8(160) = 128 lb, solving for the time t:

1 t 40 128 = 160 − 160e − t 40


 et 40 A = 4  e +c 128 − 160
1 40  = e − t 40
−160
 et 40 A = 160et 40 + c
1
 e − t 40 (et 40 A) = e − t 40 (160et 40 + c)  ln = ln[e − t 40 ]
5
 A = A(t ) = 160 + ce − t 40 1 t
 ln = −
5 40
At t = 0 mins, A = 0 lb (no salt in pure water): 1
 t = −40 ln or 40 ln 5
0 = 160 + ce −0 40 5
 c = 0 − 160  c = −160  t = 64.38 mins
Thus,
A(t ) = 160 − 160e − t 40

This document is not guaranteed by considering the items would reflect everything during your exam. But it will help you to understand,
nevertheless, on how the solution thoroughly works, provided that the methods are applied on the reference. The final answers are majority
replicated from the solution manual. Should be informed to me, typographical errors that may exist.
Problem 1 A certain population is known to be growing at a rate given by the logistic equation dx dt = x(b − ax ) . Show that the
maximum rate of growth will occur when the population is equal to half its equilibrium size, that is, when the population is b 2a .
Solution:
In order to maximize the growth rate, taking the derivative
and setting it equal to zero:
f ( x ) = x (b − ax )
 f ( x ) = bx − ax 2
 f ( x ) = b − 2ax = 0
 −2ax = −b
b
 x=
2a

Problem 5 Glucose is being fed intravenously into the bloodstream of a patient at a constant rate c grams per minute. At the same time,
the patient’s body converts the glucose and removes it from the bloodstream at a rate proportional to the amount of glucose present. If
the constant of proportionality is k, show that as time increases, the amount of glucose in the bloodstream approaches an equilibrium
value of c k.
Solution:
Let g or g(t) be the amount of glucose present at time t
and c be the constant rate.
Solving the differential equation which is expressed by
dg
= c − kg
dt
dg  Solving for the
 + kg = c 
dt  integrating factor,

 e kt (
dg
dt )
+ kg = ce kt e
E k dt = ek E dt = ekt

d kt
 e g  = ce kt
dt
 d e kt g  = ce kt dt

E E
 d e kt g  = c e kt dt
1
 e kt g = c  e kt + C
k
− kt
 e (e g) = e − kt
kt c kt
k (
e +C )
c
 g = g(t ) = + Ce − kt
k
(where C is the arbitrary constant)

When t → ,

g(t ) = lim
t → ( c
k
+ Ce− kt )
c
g= + C (0)
k
c
 g=
k

This document is not guaranteed by considering the items would reflect everything during your exam. But it will help you to understand,
nevertheless, on how the solution thoroughly works, provided that the methods are applied on the reference. The final answers are majority
replicated from the solution manual. Should be informed to me, typographical errors that may exist.
Problem 7 Suppose that the human body dissipates a drug at a rate proportional to the amount y of drug present in the bloodstream at
time t. At time t = 0 a first injection of y0 grams of the drug is made into a body that was free from that drug prior to that time.
(a) Find the amount of residual drug in the bloodstream at the end of T hours.
(b) If at time T a second injection of y0 grams is made, find the residual amount of drug at the end of 2T hours.
(c) If at the end of each time period of length T, an injection of y0 grams is made, find the residual amount of drug at the end of nT hours.
(d) Find the limiting value of the answer to part (c) as n approaches infinity.
Solution:
(a) Solving the differential equation on which the logistic model (continuation)
is expressed as For T  t  2T , the time 2T, in terms of time period, means one
dy period of time T. So,
= −kt
dt y(2T ) = y0e − kT (1 + e − kT )
1
 dy = −k dt  y(2T ) = y0 e − kT + y0e − kT e − kT
y
 y(2T ) = y0 e − kT + y0e −2 kT
 E 1
y
dy = −k dt E  y(2T ) = y0 (e− kT + e −2 kT )
 ln y = −kt + c
 e ln y = e − kt + c (c) Continuing in this way, the drug level after (n + 1)th dose is
y=e e c − kt
y(nT ) = y0 (e − kT + e −2 kT + + e − k ( n − 2)T + e − k ( n −1)T + e − knT )
 y = y(t ) = ce − kt = y0 e− kT + (e− kT )2 + + (e− kT )( n − 2) + (e− kT )( n −1) + (e− kT )n 
Since the drug dissipates over time, k has to be negative. This is the sum of the first n terms of a geometric series with
first term y0 and common ratio e− kT where e− kT  1 . The sum can
At t = 0, y = y0 :
be written as

( )
y0 = c1e−k (0)  c1 = y0
1 − e− knT
So, the first dose is y(nT ) = y0
1 − e− kT
y(t ) = y0e−kt

(d) When n → ,
Thus, for 0  t  T ,
y(T ) = y0 e − kT y = lim y0
n → ( 1 − e− knT
1 − e− kT )
1− 0
(b) With y(T) as the new initial condition, reuse the general  y = y0
1 − e− kT
solution. Shifting of coefficient is involved at which
y0
y(t ) = ce− kt  y=
1 − e− kT
 y(t ) = y(T )e − kt
 y(t ) = ( y0e− kT )e − kt

By adding previous dose, the second dose increases to drug level


to
y(t ) = y0e − kt + ( y0e − kT )e − kt
 y(t ) = y0e − kt (1 + e − kT )

This document is not guaranteed by considering the items would reflect everything during your exam. But it will help you to understand,
nevertheless, on how the solution thoroughly works, provided that the methods are applied on the reference. The final answers are majority
replicated from the solution manual. Should be informed to me, typographical errors that may exist.

You might also like